Inference Question. Leading to crack future questions...

This topic has expert replies
User avatar
Legendary Member
Posts: 934
Joined: Tue Nov 09, 2010 5:16 am
Location: AAMCHI MUMBAI LOCAL
Thanked: 63 times
Followed by:14 members
For a ten-month period, the total monthly sales of new cars within the country of Calistan remained constant. During this period the monthly sales of new cars manufactured by Marvel Automobile Company doubled, and its share of the new car market within Calistan increased correspondingly. At the end of this period, emission standards were imposed on new cars sold within Calistan. During the three months following this imposition, Marvel Automobile Company's share of the Calistan market declined substantially even though its monthly sales within Calistan
remained constant at the level reached in the last month of the ten-month period.

If the statements above are true, which one of the following CANNOT be true?

(A) The total monthly sales within Calistan of new cars by companies other than Marvel Automobile Company decreased over the three months following the imposition of the emission standards.

(B) Over the three months before the imposition of the emission standards, the combined market share of companies other than Marvel Automobile Company selling new cars in Calistan decreased.

(C) If the emission standards had not been imposed, Marvel Automobile Company would have lost an even larger share of the number of new cars sold in Calistan than, in fact, it did.

(D) A decrease in the total monthly sales of new cars within Calistan will occur if the emission standards remain in effect.

(E) Since the imposition of the emission standards, Marvel Automobile Company's average profit on each new car sold within Calistan has increased.


The OA is A.

Trust me guyzzz this question will make you ready to for a new type of question in the GMAT i.e the Must Not Be True Question Type. Just the way you have an Inference or a Must Be True question, you also have a Must Not Be True or Cannot BE True Question.

There are two ways to solve this type...

1] i.e negate each and every option and there will be 4 answer choices that will be true or infer something and 1 type will not or will not infer completely. But in that the 4 options should infer something strongly just the way you find in an inference question.

2] is that, the you are not sure about the 4 options i.e whether you can infer them or no because they genuinely must be out of scope, but you can surely and very clearly not infer a particular option or an answer choice. Just the way you get 1 answer choice independently as inferring something, in the same way you get 1 choice that simply cannot be inferred from the stimulus given.


Hence the correct here is option A. This is the Numbers and Percentages question. Solve the overall question using numbers and you will find that the option A is simply not possible even though you are not sure about the other 4 options, i.e whether you can infer them or no is not known.


This question will really help you guyzzzxzzzzz for the future difficult questions on the GMAT>\

Hope this helps!!!!! Thank You...
IT IS TIME TO BEAT THE GMAT

LEARNING, APPLICATION AND TIMING IS THE FACT OF GMAT AND LIFE AS WELL... KEEP PLAYING!!!

Whenever you feel that my post really helped you to learn something new, please press on the 'THANK' button.

User avatar
Legendary Member
Posts: 934
Joined: Tue Nov 09, 2010 5:16 am
Location: AAMCHI MUMBAI LOCAL
Thanked: 63 times
Followed by:14 members

by [email protected] » Wed May 23, 2012 4:36 am
The same question type of 'Cannot Be True' Question types can be applied to the Causal Reasoning...

Lets say the stimulus is as follows:

If A then B,

If B then C.


Answer choices:

A] If C then Z.

B] If B then P.

C] If C then A.

D] If B then A.

E] IF X then Y,


From the above answer choices the OA in this type of a question type will be D.

All the other answer choices cannot be inferred as nothing is mentioned about them so you cannot say anything about them. But when in an inference and in a Cannot Be True or a Must Not Be True, question, you have to take whatever is given in the stimulus as true and hence option D cannot be true or simply cannot be inferred. If A causes B or If then B then B cannot cause A and If B then not A is certain and hence the correct answer is D that is for sure.

This is a very important thing from the 700 level CR perspective.

Hope this thing helps you guyzzzz.......
IT IS TIME TO BEAT THE GMAT

LEARNING, APPLICATION AND TIMING IS THE FACT OF GMAT AND LIFE AS WELL... KEEP PLAYING!!!

Whenever you feel that my post really helped you to learn something new, please press on the 'THANK' button.

Legendary Member
Posts: 784
Joined: Sun Apr 03, 2011 3:51 am
Thanked: 114 times
Followed by:12 members

by patanjali.purpose » Wed May 23, 2012 10:59 am
For a ten-month period, the total monthly sales of new cars within the country of Calistan remained constant. During this period the monthly sales of new cars manufactured by Marvel Automobile Company doubled, and its share of the new car market within Calistan increased correspondingly. At the end of this period, emission standards were imposed on new cars sold within Calistan. During the three months following this imposition, Marvel Automobile Company's share of the Calistan market declined substantially even though its monthly sales within Calistan remained constant at the level reached in the last month of the ten-month period.

If the statements above are true, which one of the following CANNOT be true?
Thanks Amit.

IMO we can easily drop C/D/E:

(C) If the emission standards had not been imposed, Marvel Automobile Company would have lost an even larger share of the number of new cars sold in Calistan than, in fact, it did (We cannot say whether company would have lost larger or smaller)

(D) A decrease in the total monthly sales of new cars within Calistan will occur if the emission standards remain in effect. (may not be possible - what if company improves its processs in future. In that case it sales may not drop)

(E) Since the imposition of the emission standards, Marvel Automobile Company's average profit on each new car sold within Calistan (no idea about PROFIT in argument)

B - may be possible (market share of others may drop...we can assume some number and prove).

IMO A

User avatar
Master | Next Rank: 500 Posts
Posts: 307
Joined: Wed Feb 22, 2012 9:45 pm
Thanked: 12 times
GMAT Score:700

by Gaurav 2013-fall » Wed May 23, 2012 8:36 pm
great question! Thanks for posting. This got my head rolling.

User avatar
Master | Next Rank: 500 Posts
Posts: 307
Joined: Wed Feb 22, 2012 9:45 pm
Thanked: 12 times
GMAT Score:700

by Gaurav 2013-fall » Wed May 23, 2012 8:46 pm
finally understood this one after drawing an XY plot as well as 3 pie charts. But it took me around 10 minutes. Want to practice more of such 'head rolling problems'. Whats the source for such tough questions?

User avatar
GMAT Instructor
Posts: 1248
Joined: Thu Mar 29, 2012 2:57 pm
Location: Everywhere
Thanked: 503 times
Followed by:192 members
GMAT Score:780

by Bill@VeritasPrep » Thu May 24, 2012 12:46 pm
If Marvel's sales remained constant but its market share decreased, that means that the other companies must have sold more cars and increased their market share. This makes it pretty easy to see that A is impossible.
Join Veritas Prep's 2010 Instructor of the Year, Matt Douglas for GMATT Mondays

Visit the Veritas Prep Blog

Try the FREE Veritas Prep Practice Test

Senior | Next Rank: 100 Posts
Posts: 80
Joined: Thu Apr 12, 2012 7:00 am
Thanked: 4 times

by rajcools » Fri May 25, 2012 1:21 pm
hi without being arrogant , i think it is easy to see that A is impossible. i dont think we need to draw any graphs for it.
Marvel
Sales remain constant
Market share decrease

It cant be possible that for companies other than Marvel Market share decreased

Master | Next Rank: 500 Posts
Posts: 122
Joined: Tue May 01, 2012 6:05 am
Thanked: 3 times

by ihatemaths » Fri May 25, 2012 11:01 pm
we can easily spot A by reading the last part of the arguement carefully.

Senior | Next Rank: 100 Posts
Posts: 93
Joined: Sun Feb 22, 2015 3:47 am
Thanked: 4 times
Followed by:2 members

by bonetlobo » Wed Apr 29, 2015 9:19 am
It is quite evident that the argument is talking about ten months period and three months after this period. It is also noticeable that emission standards were imposed on new cars at the "end" of the 10 months period.

Apart from A, the other options "can" be true.

B: Over the three months before the imposition of the emission standards. This means, we are talking about the months 8, 9 and 10. Notice that this was the period when Marvel Automobile Company was doing great. So, it is definitely possible that combined market share of companies "other" than Marvel Automobile Company selling new cars in Calistan decreased.

C: We have absolutely no information about it from the argument; in fact, we don't even know for sure whether imposition of the emission standards was the "reason" for Marvel Automobile Company's share of the Calistan market declining substantially.

D: Again, no information about this in the argument. "If" the emission standards remain in effect, what will or will not happen, who knows?

E: No information about "profit" is provided. The most we know is about "sales", not "profit"